Diễn Đàn MathScopeDiễn Đàn MathScope
  Diễn Đàn MathScope
Ghi Danh Hỏi/Ðáp Thành Viên Social Groups Lịch Ðánh Dấu Ðã Ðọc

Go Back   Diễn Đàn MathScope > Sơ Cấp > Việt Nam và IMO > 2013

News & Announcements

Ngoài một số quy định đã được nêu trong phần Quy định của Ghi Danh , mọi người tranh thủ bỏ ra 5 phút để đọc thêm một số Quy định sau để khỏi bị treo nick ở MathScope nhé !

* Nội quy MathScope.Org

* Một số quy định chung !

* Quy định về việc viết bài trong diễn đàn MathScope

* Nếu bạn muốn gia nhập đội ngũ BQT thì vui lòng tham gia tại đây

* Những câu hỏi thường gặp

* Về việc viết bài trong Box Đại học và Sau đại học


Trả lời Gởi Ðề Tài Mới
 
Ðiều Chỉnh Xếp Bài
Old 24-07-2013, 11:19 PM   #1
novae
+Thành Viên Danh Dự+
 
novae's Avatar
 
Tham gia ngày: Jul 2010
Đến từ: Event horizon
Bài gởi: 2,453
Thanks: 53
Thanked 3,057 Times in 1,288 Posts
[IMO 2013] Bài 5 - Phương trình hàm

Kí hiệu $\mathbb{Q}_{>0}$ là tập hợp các số hữu tỉ dương. Cho $f : \mathbb{Q}_{>0} \to \mathbb{R}$ là hàm số thỏa mãn ba điều kiện sau:
  1. với mọi $x,y \in \mathbb{Q}_{>0}$, ta có $f(x)f(y) \ge f(xy)$;
  2. với mọi $x,y \in \mathbb{Q}_{>0}$, ta có $f(x+y) \ge f(x)+f(y)$;
  3. tồn tại số hữu tỉ $a>1$ sao cho $f(a)=a$.
Chứng minh rằng $f(x)=x$ với mọi $x \in \mathbb{Q}_{>0}$.
[RIGHT][I][B]Nguồn: MathScope.ORG[/B][/I][/RIGHT]
 
__________________
M.

thay đổi nội dung bởi: novae, 25-07-2013 lúc 10:03 AM
novae is offline   Trả Lời Với Trích Dẫn
The Following User Says Thank You to novae For This Useful Post:
hieu1411997 (26-07-2013)
Old 25-07-2013, 07:32 AM   #2
hakudoshi
+Thành Viên+
 
hakudoshi's Avatar
 
Tham gia ngày: Feb 2012
Đến từ: vật chất->sự sống->tư duy->cảm xúc->???
Bài gởi: 210
Thanks: 102
Thanked 179 Times in 90 Posts
Trích:
Nguyên văn bởi novae View Post
[*]với mọi $x,y \in \mathbb{Q}_{>0}$, ta có $f(x)+f(y) \ge f(x)+f(y)$;
Check lại hộ em đoạn này
[RIGHT][I][B]Nguồn: MathScope.ORG[/B][/I][/RIGHT]
 
__________________
Touch me touch me, don't be shy
I'm in charge like a G.U.Y.
I'll lay down face up this time
Under you like a G.U.Y.
hakudoshi is offline   Trả Lời Với Trích Dẫn
Old 25-07-2013, 09:33 AM   #3
hoangnam94
+Thành Viên+
 
Tham gia ngày: Jan 2010
Đến từ: Nhơn Trạch-Đồng Nai
Bài gởi: 244
Thanks: 105
Thanked 40 Times in 21 Posts
Trích:
Nguyên văn bởi hakudoshi View Post
Check lại hộ em đoạn này
Anh novae gõ nhầm một chút thôi mà bạn. Bạn sửa lại là
ii) với mọi $x,y \in \mathbb{Q}_{>0}$ thì $f(x+y) \geq f(x)+f(y)$
[RIGHT][I][B]Nguồn: MathScope.ORG[/B][/I][/RIGHT]
 
__________________

thay đổi nội dung bởi: hoangnam94, 25-07-2013 lúc 09:39 AM
hoangnam94 is offline   Trả Lời Với Trích Dẫn
Old 25-07-2013, 10:04 AM   #4
novae
+Thành Viên Danh Dự+
 
novae's Avatar
 
Tham gia ngày: Jul 2010
Đến từ: Event horizon
Bài gởi: 2,453
Thanks: 53
Thanked 3,057 Times in 1,288 Posts
Hôm qua buồn ngủ quá nên gõ nhầm Anh sửa lại rồi nhé
[RIGHT][I][B]Nguồn: MathScope.ORG[/B][/I][/RIGHT]
 
__________________
M.
novae is offline   Trả Lời Với Trích Dẫn
The Following User Says Thank You to novae For This Useful Post:
hoangnam94 (25-07-2013)
Old 25-07-2013, 02:24 PM   #5
doxuantung97
+Thành Viên+
 
Tham gia ngày: Aug 2012
Đến từ: Toán 1 K46 Chuyên Sư Phạm
Bài gởi: 49
Thanks: 19
Thanked 24 Times in 12 Posts
Trích:
Nguyên văn bởi novae View Post
Hôm qua buồn ngủ quá nên gõ nhầm Anh sửa lại rồi nhé
Sặc! Trời ơi! Hôm qua em mất hơn tiếng làm cái bài này! Nhưng tại anh gõ sai nên em đoán là $f(x)+f(y) \geq f(x+y)$.
[RIGHT][I][B]Nguồn: MathScope.ORG[/B][/I][/RIGHT]
 
doxuantung97 is offline   Trả Lời Với Trích Dẫn
Old 25-07-2013, 02:25 PM   #6
chungmathkb
+Thành Viên+
 
Tham gia ngày: Sep 2009
Bài gởi: 18
Thanks: 0
Thanked 37 Times in 9 Posts
Trích:
Nguyên văn bởi novae View Post
Kí hiệu $\mathbb{Q}_{>0}$ là tập hợp các số hữu tỉ dương. Cho $f : \mathbb{Q}_{>0} \to \mathbb{R}$ là hàm số thỏa mãn ba điều kiện sau:
  1. với mọi $x,y \in \mathbb{Q}_{>0}$, ta có $f(x)f(y) \ge f(xy)$;
  2. với mọi $x,y \in \mathbb{Q}_{>0}$, ta có $f(x+y) \ge f(x)+f(y)$;
  3. tồn tại số hữu tỉ $a>1$ sao cho $f(a)=a$.
Chứng minh rằng $f(x)=x$ với mọi $x \in \mathbb{Q}_{>0}$.
Trong (i), lấy $x=1$ và $y=a$, ta được
$$f(1)f(a) \ge f(a) \Rightarrow af(1) \ge a \Rightarrow f(1) \ge 1.$$
Từ (2) suy ra với $n\in\mathbb Z, n\ge1$, ta có
$f(nx) \ge nf(x),\,\forall x \in {\mathbb Q_{ > 0}}$, suy ra
\[f(n) \ge nf(1) \ge n \Rightarrow f(n) \ge n,\,\forall n \in {\mathbb N^*}.\tag{1}\]
Ta có $ f(q)f(\dfrac{p}{q})\mathop \ge \limits^{\text{do}\,\,(i)} f(p)\mathop \ge \limits^{\text{do}\,\,(1)} p\,\,\,(p,q \in {\mathbb N^*}).$ Từ
đây suy ra
\[f(x) > 0,\,\forall x \in {\mathbb Q_{ > 0}}.\tag{2}\]
Từ (2) và (ii) suy ra $f$ là hàm tăng nghiêm ngặt trên ${\mathbb Q_{ > 0}}$.
Với $q$ là số hữu tỉ, $q>1$, ta có $f(q) \ge f([q]) \ge [q] > q - 1$.

$\bullet$ Chứng minh: $ f(q)\ge q $, với $q$ là số hữu tỉ lớn hơn 1.
Giả sử tồn tại $q\in\mathbb Q$, $q>1$ sao cho $f(q)<q$, khi đó $ f(q) = q-\varepsilon $, với $0 < \varepsilon < 1$.
Ta có
\[{(q - \varepsilon )^n} = f{(q)^n}\mathop \ge \limits^{\text{do}\,\,(i)} f({q^n}) > {q^n} - 1.\]
Do đó
${\left( {\dfrac{{q - \varepsilon }}{q}} \right)^n} > 1 - \dfrac{1}{{{q^n}}},\,\forall n = 1,2,\dots$
Từ đây cho $n\to+\infty$, ta được $0\ge 1$, vô lí. Vậy với mọi
$q$ là số hữu tỉ lớn hơn 1, ta có $ f(q)\ge q $.

$\bullet$ Chứng minh: $ f(q)= q $, với $q$ là số hữu tỉ, $q\ge 1$.
Chọn số nguyên dương $n$ đủ lớn sao cho $ a^n-q > 1 $. Khi đó
$${a^n} = f{(a)^n} \ge f({a^n}) = f({a^n} - q + q) \ge f({a^n} - q) + f(q) \ge {a^n} - q + q = {a^n}.$$
Suy ra $ f(q)= q $, với $q$ là số hữu tỉ, $q\ge 1$. (3)

$\bullet$ Giả sử $q \in {\mathbb Q_{ > 0}}$ và $q<1$. Khi đó $\dfrac{1}{q} > 1$. Ta có
$$f(q)f(\frac{1}{q}) \ge f(1) \Rightarrow f(q)\frac{1}{q} \ge f(1) \Rightarrow f(q)\frac{1}{q} \ge 1 \Rightarrow f(q) \ge q.$$
Vậy $f(q) \ge q,\,\forall q \in {\mathbb Q_{ > 0}}$.
Giả sử tồn tại số hữu tỉ $q\in(0;1)$ sao cho $f(q)>q$. Khi đó
\[f(nq)\mathop \ge \limits^{\text{do}\,\,(ii)} nf(q) > nq,\,\forall n = 2,3,\dots\tag{4}\]
Chọn số nguyên dương $n$ đủ lớn sao cho $nq>1$, khi đó từ (4) ta có $f(nq)>nq$, điều này mâu thuẫn với (3).
Do đó, với mọi số hữu tỉ $q\in(0;1)$, ta có $f(q)=q$. (5)

Từ (3) và (5) suy ra $f(x) = x,\,\forall x \in {\mathbb Q_{ > 0}}$, điều phải chứng minh.
[RIGHT][I][B]Nguồn: MathScope.ORG[/B][/I][/RIGHT]
 

thay đổi nội dung bởi: magician_14312, 25-07-2013 lúc 09:34 PM
chungmathkb is offline   Trả Lời Với Trích Dẫn
Old 25-07-2013, 11:12 PM   #7
quocbaoct10
+Thành Viên Danh Dự+
 
quocbaoct10's Avatar
 
Tham gia ngày: Oct 2012
Đến từ: THPT chuyên Lê Quý Đôn-Nha Trang-Khánh Hòa
Bài gởi: 539
Thanks: 292
Thanked 365 Times in 217 Posts
Em có 1 cách làm như thế này, tuy hơi dài nhưng khá đơn giản.
Dễ chứng minh $f(1) \ge 1$.
ta chứng minh:$(f(a))^2 \ge a^2.f(1)$. Thật vậy, từ $f(x).f(y)=f(x.y)$ (1), ta thay:$x=y=a$, được: $(f(a))^2 \ge f(a^2)$. Mà từ $f(x+y) \ge f(x)+f(y) $ (2), ta có: $f(a^2) \ge f(a^{2}-1) +f(1) \ge f(a^{2}-2)+2f(1) \ge ... \ge a^{2}.f(1)$ suy ra $(f(a))^2=a^2 \ge a^2.f(1) \Rightarrow f(1) \le 1 $. Mà ta đã chứng minh $f(1) \ge 1$ nên $f(1)=1$. Xét TH $a$ không nguyên, khi đó: $a^2=f(a)^2 \ge f(a^2) \ge f(a^2-1)+f(1) \ge a^2-1+1=a^2$ nên $f(1)=1$ . Từ đó dễ chứng minh bằng quy nạp rằng $f(n)=n$ với mọi $n \in \mathbb N*$. Giờ ta chứng minh $f(\frac{1}{n})=\frac{1}{n}$.
THật vậy: $1=f(1)=f(n.\frac{1}{n}) \le f(n).f(\frac{1}{n}) \Leftrightarrow f(\frac{1}{n}) \ge \frac{1}{n}$, tương tự, ta chứng minh được: $f(\frac{m}{n}) \ge \frac{m}{n}$. Mà $1=f(1)=f(\frac{n}{n}) \ge f(\frac{n-1}{n})+f(\frac{1}{n}) \ge f(\frac{n-2}{n}) + 2.f(\frac{1}{n}) \ge ... \ge n.f(\frac{1}{n}) \Rightarrow f(\frac{1}{n}) \le \frac{1}{n}$ mà ta cũng đã chứng minh $f(\frac{1}{n}) \ge \frac{1}{n}$ nên $f(\frac{1}{a})=\frac{1}{a}$ với mọi $n \in \mathbb N*$.
Giờ ta chứng minh $f(\frac{m}{n})=\frac{m}{n}$.
Ta có: $f(\frac{m}{n}). f(\frac{n}{m}) \ge f(1) $, mà $f(\frac{m}{n}). f(\frac{n}{m}) \le f(m).f(\frac{1}{n}).f(n).f(\frac{1}{m})= 1$. Từ đấy suy ra $f(\frac{m}{n}). f(\frac{n}{m})=1$, mà $f(\frac{m}{n}) \ge \frac{m}{n}, f(\frac{n}{m}) \ge \frac{n}{m}$ nên $f(\frac{m}{n})= \frac{n}{m}, f(\frac{n}{m}) = \frac{n}{m}$ với mọi $n;m \in \mathbb N*$.
[RIGHT][I][B]Nguồn: MathScope.ORG[/B][/I][/RIGHT]
 
__________________
i'll try my best.

thay đổi nội dung bởi: quocbaoct10, 26-07-2013 lúc 12:35 AM
quocbaoct10 is offline   Trả Lời Với Trích Dẫn
Old 25-07-2013, 11:33 PM   #8
12121993
+Thành Viên+
 
Tham gia ngày: Feb 2012
Bài gởi: 81
Thanks: 23
Thanked 70 Times in 41 Posts
Trích:
Nguyên văn bởi quocbaoct10 View Post
Từ đó dễ chứng minh bằng quy nạp rằng $f(n)=n$ với mọi $n \in \mathbb N*$.
THật vậy: $1=f(1)=f(n.\frac{1}{n}) \ge f(n)$.$f(\frac{1}{n}) \Leftrightarrow f(\frac{1}{n}) \ge \frac{1}{n}$
Bạn thử viết rõ cách chứng minh $f(n)=n$ ???
Dòng tô đỏ nhầm dấu nhé.
[RIGHT][I][B]Nguồn: MathScope.ORG[/B][/I][/RIGHT]
 

thay đổi nội dung bởi: 12121993, 25-07-2013 lúc 11:37 PM
12121993 is offline   Trả Lời Với Trích Dẫn
Old 25-07-2013, 11:37 PM   #9
toansocaplqd
+Thành Viên+
 
Tham gia ngày: Jun 2013
Đến từ: TP. Phan Rang-Tháp Chàm, tỉnh Ninh Thuận
Bài gởi: 82
Thanks: 69
Thanked 10 Times in 9 Posts
Trích:
Nguyên văn bởi quocbaoct10 View Post
Em có 1 cách làm như thế này, tuy hơi dài nhưng khá đơn giản.
Dễ chứng minh $f(1) \ge 1$.
ta chứng minh:$(f(a))^2 \ge a^2.f(1)$. Thật vậy, từ $f(x).f(y)=f(x.y)$ (1), ta thay:$x=y=a$, được: $(f(a))^2 \ge f(a^2)$. Mà từ $f(x+y) \ge f(x)+f(y) $ (2), ta có: $f(a^2) \ge f(a^{2}-1) +f(1) \ge f(a^{2}-2)+2f(1) \ge ... \ge a^{2}.f(1)$ suy ra $(f(a))^2=a^2 \ge a^2.f(1) \Rightarrow f(1) \le 1 $. Mà ta đã chứng minh $f(1) \ge 1$ nên $f(1)=1$. Từ đó dễ chứng minh bằng quy nạp rằng $f(n)=n$ với mọi $n \in \mathbb N*$. Giờ ta chứng minh $f(\frac{1}{n})=\frac{1}{n}$.
THật vậy: $1=f(1)=f(n.\frac{1}{n}) \ge f(n).f(\frac{1}{n}) \Leftrightarrow f(\frac{1}{n}) \ge \frac{1}{n}$, tương tự, ta chứng minh được: $f(\frac{m}{n}) \ge \frac{m}{n}$. Mà $1=f(1)=f(\frac{n}{n}) \ge f(\frac{n-1}{n})+f(\frac{1}{n}) \ge f(\frac{n-2}{n}) + 2.f(\frac{1}{n}) \ge ... \ge n.f(\frac{1}{n}) \Rightarrow f(\frac{1}{n}) \le \frac{1}{n}$ mà ta cũng đã chứng minh $f(\frac{1}{n}) \ge \frac{1}{n}$ nên $f(\frac{1}{a})=\frac{1}{a}$ với mọi $n \in \mathbb N*$.
Giờ ta chứng minh $f(\frac{m}{n})=\frac{m}{n}$.
Ta có: $f(\frac{m}{n}). f(\frac{n}{m}) \ge f(1) $, mà $f(\frac{m}{n}). f(\frac{n}{m}) \le f(m).f(\frac{1}{n}).f(n).f(\frac{1}{m})= 1$. Từ đấy suy ra $f(\frac{m}{n}). f(\frac{n}{m})=1$, mà $f(\frac{m}{n}) \ge \frac{m}{n}, f(\frac{n}{m}) \ge \frac{n}{m}$ nên $f(\frac{m}{n})= \frac{n}{m}, f(\frac{n}{m}) = \frac{n}{m}$ với mọi $n;m \in \mathbb N*$.
Bạn xem lại phần đầu vì $f(x)$ nhận giá trị trên $R$.
[RIGHT][I][B]Nguồn: MathScope.ORG[/B][/I][/RIGHT]
 
toansocaplqd is offline   Trả Lời Với Trích Dẫn
Old 26-07-2013, 12:07 AM   #10
quocbaoct10
+Thành Viên Danh Dự+
 
quocbaoct10's Avatar
 
Tham gia ngày: Oct 2012
Đến từ: THPT chuyên Lê Quý Đôn-Nha Trang-Khánh Hòa
Bài gởi: 539
Thanks: 292
Thanked 365 Times in 217 Posts
Trích:
Nguyên văn bởi 12121993 View Post
Bạn thử viết rõ cách chứng minh $f(n)=n$ ???
Dòng tô đỏ nhầm dấu nhé.
có $f(1)=1$. Ta giả sử nó đúng tới $f(n)$ (n>1), khi đó $f(n)=n$. Ta chứng minh $f(n+1) \ge n+1$. Thật vậy $f(n+1) \ge f(n)+f(1) \ge f(n-1)+2f(1) \ge ... \ge (n+1).f(1)=n+1$. Nếu $n+1$ không nguyên tố, đặt $n+1=b.c$ suy ra $b;c < n$ vậy nên theo giả thiết quy nạp thì $f(b)=b, f(c)=c$. Khi đó: $n+1=b.c=f(b).f(c) \ge f(b.c)=f(n+1)$ suy ra $f(n+1)=n+1$. Nếu $n+1$ nguyên tố thì $n+1=2k+1$. xét số $n+2=2(k+1)$. Lúc này, $k+1<n$ và $2<n$ nên từ đấy, ta dễ cm :$f(n+2)=n+2$. Ta có: $f(n+2) \ge f(n+1)+1 \Rightarrow f(n+1) \le n+1$ nên suy ra $f(n+1)=n+1$ (đpcm).
Trích:
Nguyên văn bởi toansocaplqd View Post
Bạn xem lại phần đầu vì $f(x)$ nhận giá trị trên $R$.
khi mình đã cm được $f(\frac{m}{n})=\frac{m}{n}$ thì lúc này tập R không còn ý nghĩa nữa vì $\frac{m}{n}$ là tập số hữu tỉ thì kéo theo $f(\frac{m}{n})$ là số hữu tỉ.

[RIGHT][I][B]Nguồn: MathScope.ORG[/B][/I][/RIGHT]
 
__________________
i'll try my best.
quocbaoct10 is offline   Trả Lời Với Trích Dẫn
Old 26-07-2013, 12:52 AM   #11
12121993
+Thành Viên+
 
Tham gia ngày: Feb 2012
Bài gởi: 81
Thanks: 23
Thanked 70 Times in 41 Posts
Trích:
Nguyên văn bởi quocbaoct10 View Post
có $f(1)=1$. Ta giả sử nó đúng tới $f(n)$ (n>1), khi đó $f(n)=n$. Ta chứng minh $f(n+1) \ge n+1$. Thật vậy $f(n+1) \ge f(n)+f(1) \ge f(n-1)+2f(1) \ge ... \ge (n+1).f(1)=n+1$. Nếu $n+1$ không nguyên tố, đặt $n+1=b.c$ suy ra $b;c < n$ vậy nên theo giả thiết quy nạp thì $f(b)=b, f(c)=c$. Khi đó: $n+1=b.c=f(b).f(c) \ge f(b.c)=f(n+1)$ suy ra $f(n+1)=n+1$. Nếu $n+1$ nguyên tố thì $n+1=2k+1$. xét số $n+2=2(k+1)$. Lúc này, $k+1<n$ và $2<n$ nên từ đấy, ta dễ cm :$f(n+2)=n+2$. Ta có: $f(n+2) \ge f(n+1)+1 \Rightarrow f(n+1) \le n+1$ nên suy ra $f(n+1)=n+1$ (đpcm).

khi mình đã cm được $f(\frac{m}{n})=\frac{m}{n}$ thì lúc này tập R không còn ý nghĩa nữa vì $\frac{m}{n}$ là tập số hữu tỉ thì kéo theo $f(\frac{m}{n})$ là số hữu tỉ.
Bước cơ sở của bạn là chưa có, bạn phải kiểm tra với $n=2$.
[RIGHT][I][B]Nguồn: MathScope.ORG[/B][/I][/RIGHT]
 
12121993 is offline   Trả Lời Với Trích Dẫn
Old 26-07-2013, 07:32 AM   #12
123456
+Thành Viên+
 
123456's Avatar
 
Tham gia ngày: May 2008
Đến từ: Ha Noi
Bài gởi: 709
Thanks: 13
Thanked 613 Times in 409 Posts
Trích:
Nguyên văn bởi novae View Post
Kí hiệu $\mathbb{Q}_{>0}$ là tập hợp các số hữu tỉ dương. Cho $f : \mathbb{Q}_{>0} \to \mathbb{R}$ là hàm số thỏa mãn ba điều kiện sau:
  1. với mọi $x,y \in \mathbb{Q}_{>0}$, ta có $f(x)f(y) \ge f(xy)$;
  2. với mọi $x,y \in \mathbb{Q}_{>0}$, ta có $f(x+y) \ge f(x)+f(y)$;
  3. tồn tại số hữu tỉ $a>1$ sao cho $f(a)=a$.
Chứng minh rằng $f(x)=x$ với mọi $x \in \mathbb{Q}_{>0}$.
Chọn $x=a, y=1$ trong $(i)$ ta được $f(1)\geq 1$, do đó từ $(ii)$ suy ra $f(m) \geq m$ với mọi số tự nhiên $m$.

Với mọi $x\in \mathbb{Q}_{>0}$, tồn tại các số tự nhiên $m, n$ sao cho $x = m/n$, khi đó ta có $f(n) f(x) \geq f(nx) = f(m) \geq m$ hay là $f(x) \geq m/f(n) >0$. Vậy $f$ là hàm dương.

Ta chứng minh $f(x) =x$ với $x \in \mathbb{Q}_{>0}$:

Với mọi số tự nhiên $n$, tồn tại số tự nhiên $k_n$ sao cho $a^n = k_n x + r_n$ với $0\leq r_n < x$. Sử dụng $(i)$, $(ii)$ và tính dương của hàm $f$ ta có
$$a^n = f(a)^n\geq f(a^n)=f(k_nx+r_n) \geq k_nf(x) +f(r_n)\geq k_n f(x).$$
Do đó
$$f(x) \leq x + \frac{r_n}{k_n} \leq x + \frac{x}{k_n},$$
cho $n$ ra vô cùng (vì $a>1$ nên $a^n$ ra vô cùng, do đó $k_n$ cũng ra vô cùng) ta được $f(x) \leq x$.

Với mọi số tự nhiên $n$, tồn tại số tự nhiên $k_n$ sao cho $nx = k_n a+ r_n$ với $0\leq r_n < a$. Sử dụng kết quả vừa chứng minh, $(i)$, $(ii)$ và tính dương của hàm $f$ ta có
$$nf(x) \geq f(n) f(x) \geq f(nx) = f(k_n a+r_n) \geq k_n f(a) + f(r_n)\geq k_n a.$$
Do đó
$$f(x) \geq \frac{k_n a}n = x -\frac{r_n}{n} \geq x -\frac{x}{n},$$
Cho $n$ ra vô cùng ta được $f(x) \geq x$.

Vậy $f(x) = x$.

Ps: Một lí luận khác rằng từ trên ta có $f(x) \leq x$ với mọi $x\in \mathbb{Q}_{>0}$. Do đó $f(m) = m$ với mọi số tự nhiên $m$. Dùng $(i)$ và $(ii)$ suy ra $f(\frac1m) = \frac1m$ với mọi $m$, từ đây suy ra dpcm.
------------------------------
Trích:
Nguyên văn bởi quocbaoct10 View Post
có $f(1)=1$. Ta giả sử nó đúng tới $f(n)$ (n>1), khi đó $f(n)=n$. Ta chứng minh $f(n+1) \ge n+1$. Thật vậy $f(n+1) \ge f(n)+f(1) \ge f(n-1)+2f(1) \ge ... \ge (n+1).f(1)=n+1$. Nếu $n+1$ không nguyên tố, đặt $n+1=b.c$ suy ra $b;c < n$ vậy nên theo giả thiết quy nạp thì $f(b)=b, f(c)=c$. Khi đó: $n+1=b.c=f(b).f(c) \ge f(b.c)=f(n+1)$ suy ra $f(n+1)=n+1$. Nếu $n+1$ nguyên tố thì $n+1=2k+1$. xét số $n+2=2(k+1)$. Lúc này, $k+1<n$ và $2<n$ nên từ đấy, ta dễ cm :$f(n+2)=n+2$. Ta có: $f(n+2) \ge f(n+1)+1 \Rightarrow f(n+1) \le n+1$ nên suy ra $f(n+1)=n+1$ (đpcm).
Lí luận qui nạp ở đây bị hổng khi $n=2$ là số nguyên tố nhưng là số chẵn. Nếu chứng minh được $f(2) =2$ thì bài toán được giải vì nếu $f(2) = 2$ thì $f(2^n) = 2^n$ với mọi $n$. Sử dụng qui nạp ngược nếu $f(k) = k$ thì $f(k-1) = k-1$, khi đó ta được $f(m) = m$ với mọi số tự nhiên $m$, ....
[RIGHT][I][B]Nguồn: MathScope.ORG[/B][/I][/RIGHT]
 

thay đổi nội dung bởi: 123456, 26-07-2013 lúc 07:49 AM Lý do: Tự động gộp bài
123456 is offline   Trả Lời Với Trích Dẫn
The Following User Says Thank You to 123456 For This Useful Post:
bboy114crew (26-07-2013)
Old 26-07-2013, 09:24 AM   #13
quocbaoct10
+Thành Viên Danh Dự+
 
quocbaoct10's Avatar
 
Tham gia ngày: Oct 2012
Đến từ: THPT chuyên Lê Quý Đôn-Nha Trang-Khánh Hòa
Bài gởi: 539
Thanks: 292
Thanked 365 Times in 217 Posts
Trích:
Nguyên văn bởi 123456 View Post
Lí luận qui nạp ở đây bị hổng khi $n=2$ là số nguyên tố nhưng là số chẵn. Nếu chứng minh được $f(2) =2$ thì bài toán được giải vì nếu $f(2) = 2$ thì $f(2^n) = 2^n$ với mọi $n$. Sử dụng qui nạp ngược nếu $f(k) = k$ thì $f(k-1) = k-1$, khi đó ta được $f(m) = m$ với mọi số tự nhiên $m$, ....
vì $a>1$ nên tồn tại số k nguyên dương đủ lớn để $a^k > 2$. Khi đó, ta có: $a^{k}=f(a)^k \ge f(a^k)=f(a^{k}-2+2) \ge f(a^{k}-2)+f(2) \ge a^k-2+2=a^k$ nên $f(a^{k}-2)+f(2)=a^k$, mà $f(a^{k}-2) \ge a^{k}-2, f(2) \ge 2$ nên $f(2)=2$.
[RIGHT][I][B]Nguồn: MathScope.ORG[/B][/I][/RIGHT]
 
__________________
i'll try my best.

thay đổi nội dung bởi: quocbaoct10, 26-07-2013 lúc 04:49 PM
quocbaoct10 is offline   Trả Lời Với Trích Dẫn
Old 26-07-2013, 08:14 PM   #14
hansongkyung
+Thành Viên+
 
Tham gia ngày: Jan 2012
Đến từ: Han Tae Woong - IMO 1998
Bài gởi: 493
Thanks: 109
Thanked 417 Times in 241 Posts
Gửi tin nhắn qua Yahoo chát tới hansongkyung
Trích:
Nguyên văn bởi quocbaoct10 View Post
Xét TH $a$ không nguyên, khi đó: $a^2=f(a)^2 \ge f(a^2) \ge f(a^2-1)+f(1) \ge a^2-1+1=a^2$ nên $f(1)=1$ . Từ đó dễ chứng minh bằng quy nạp rằng $f(n)=n$ với mọi $n \in \mathbb N*$. Giờ ta chứng minh $f(\frac{1}{n})=\frac{1}{n}$.
Tại sao lại có $f(a^2 -1) \ge a^2 -1$ Hình như chỉ chứng minh được $f(n) \ge n$ với $\forall n \in \mathbb{Z}^+$ chứ không đúng với số hữu tỉ.
[RIGHT][I][B]Nguồn: MathScope.ORG[/B][/I][/RIGHT]
 
hansongkyung is offline   Trả Lời Với Trích Dẫn
Old 26-07-2013, 09:58 PM   #15
linh1997
+Thành Viên+
 
Tham gia ngày: Nov 2011
Bài gởi: 133
Thanks: 27
Thanked 31 Times in 15 Posts
Em có cách này không biết có sai gì không ?
từ (1) và (3) suy ra $f(ax) \le af(x)$ mà từ (2) lại có $f(ax) \ge af(x)$ với mọi a hữu tỷ dương suy ra $f(ax) = af(x)$ từ đây, ta có : f(x) = ax (với a = f(1))
thử lại ta có a = 1
[RIGHT][I][B]Nguồn: MathScope.ORG[/B][/I][/RIGHT]
 
__________________
lúc khó khăn nhất là lúc thành công không còn xa nữa
linh1997 is offline   Trả Lời Với Trích Dẫn
Trả lời Gởi Ðề Tài Mới

Bookmarks

Ðiều Chỉnh
Xếp Bài

Quuyền Hạn Của Bạn
You may not post new threads
You may not post replies
You may not post attachments
You may not edit your posts

BB code is Mở
Smilies đang Mở
[IMG] đang Mở
HTML đang Tắt

Chuyển đến


Múi giờ GMT. Hiện tại là 12:01 PM.


Powered by: vBulletin Copyright ©2000-2024, Jelsoft Enterprises Ltd.
Inactive Reminders By mathscope.org
[page compression: 108.41 k/124.81 k (13.14%)]